Diễn Đàn MathScopeDiễn Đàn MathScope
  Diễn Đàn MathScope
Ghi Danh Hỏi/Ðáp Community Lịch

Go Back   Diễn Đàn MathScope > Sơ Cấp > Tài Liệu > Đề Thi > Đề Chọn Đội Tuyển Trường

News & Announcements

Ngoài một số quy định đã được nêu trong phần Quy định của Ghi Danh , mọi người tranh thủ bỏ ra 5 phút để đọc thêm một số Quy định sau để khỏi bị treo nick ở MathScope nhé !

* Nội quy MathScope.Org

* Một số quy định chung !

* Quy định về việc viết bài trong diễn đàn MathScope

* Nếu bạn muốn gia nhập đội ngũ BQT thì vui lòng tham gia tại đây

* Những câu hỏi thường gặp

* Về việc viết bài trong Box Đại học và Sau đại học


Trả lời Gởi Ðề Tài Mới
 
Ðiều Chỉnh Xếp Bài
Old 15-10-2012, 07:31 PM   #1
Highschoolmath
Moderator
 
Highschoolmath's Avatar
 
Tham gia ngày: Apr 2008
Đến từ: Hàm Dương-Đại Tần
Bài gởi: 698
Thanks: 247
Thanked 350 Times in 224 Posts
Đề chọn đội tuyển quốc gia 2012-2013 trường chuyên ĐHSP HN ngày thi thứ nhất

Gõ lại cái đề cho tử tế:
Câu 1: Cho các số thực dương $x,y,z $ thõa mãn $x+y+z=3 $. Chứng minh rằng:
$\frac{4x+5}{x^3+xy^2+3xyz}+\frac{4y+5}{y^3+yz^2+3x yz}+\frac{4z+5}{z^3+zx^2+3xyz} \geq \frac{162}{x^2+y^2+z^2+27} $.

Câu 2: Cho tứ giác $ABCD $ có $AB, CD $ cắt nhau ở $P $. Gọi $E $ là điểm đối xứng với $D $ qua $P $; $(O_1) $ là đường tròn đi qua $A,B $; $(O_2) $ là đường tròn đi qua $C, E $; $O $ là trung điểm $O_1O_2 $. Giả sử $(O_1), (O_2) $ cắt nhau ở $X,Y $. Chứng minh rằng $O $ là tâm đường tròn ngoại tiếp tam giác $PXY $ khi và chỉ khi tứ giác $ABCD $ nội tiếp.

Câu 3: Cho $p=6^{2^n}+1 $ là một số nguyên tố, trong đó $n $ là một số nguyên dương.
a/ Chứng minh rằng $6^{p-1}-1 $ không chia hết cho $p^2 $.
b/ Giả sử $x,y $ là hai số thuộc tập hợp $\left\{ 2,3,\ldots,\frac{p-1}{2} \right\} $ và thỏa mãn điều kiện $x^2 \equiv 36y^2 \pmod{p} $. Chứng minh rằng tồn tại duy nhất một số nguyên dương $k $ sao cho $x^{2^k}+y^{2^k} $ là hợp số và chia hết cho $p $.

Câu 4: Cho 2012 số thực $x_1,x_2,\ldots,x_{2012} $ đôi một khác nhau thõa mãn điều kiện $\sum_{i=1}^{2012}x_i=0 $. Chứng minh rằng tồn tại $C_{2011}^{502} $ bộ chỉ số $(i_1,i_2,\ldots,i_{503}) $ thõa mãn:
i/ $1 \leq i_1< i_2<\cdots<i_{503} \leq 2012 $
ii/ $x_{i_1}+x_{i_2} + \cdots + x_{i_{503}} \geq 0 $.
-------------------------------------Hết--------------------------------
[RIGHT][I][B]Nguồn: MathScope.ORG[/B][/I][/RIGHT]
 
__________________
As long as I live, I shall think only of the Victory......................

thay đổi nội dung bởi: Highschoolmath, 16-10-2012 lúc 10:05 AM
Highschoolmath is offline   Trả Lời Với Trích Dẫn
The Following 11 Users Say Thank You to Highschoolmath For This Useful Post:
cauchy_schwarz (16-10-2012), dep_kom_n (15-10-2012), ghost95 (29-10-2012), mathmath123 (29-10-2012), ngocson_dhsp (15-10-2012), pexea12 (16-10-2012), pqhoai (15-10-2012), quykhtn (15-10-2012), TNP (15-10-2012), tungk45csp (15-10-2012), zớt (15-10-2012)
Old 15-10-2012, 10:19 PM   #2
tikita
Administrator

 
Tham gia ngày: Jun 2012
Bài gởi: 157
Thanks: 2
Thanked 84 Times in 53 Posts
Trích:
Nguyên văn bởi Highschoolmath View Post
Gõ lại cái đề cho tử tế:
Câu 1: Cho các số thực dương $x,y,z $ thõa mãn $x+y+z=3 $. Chứng minh rằng:
$\frac{4x+5}{x^3+xy^2+3xyz}+\frac{4y+5}{y^3+yz^2+3x yz}+\frac{4z+5}{z^3+zx^2+3xyz} \geq \frac{162}{x^2+y^2+z^2+27} $.
Bất đẳng thức đầu bài được suy ra từ các bất đẳng thức sau:
  1. $\sum \dfrac{4+\frac{5}{x}}{x^2+y^2+3yz}\geq\dfrac{(\sum \sqrt{4+\frac{5}{x}})^2}{2x^2+2y^2+2z^2+3xy+3yz+3x z}$
  2. $2x^2+2y^2+2z^2+3xy+3yz+3xz=\dfrac{1}{2}(x^2+y^2+z ^2+27)$
  3. $\sqrt{4+\frac{5}{x}}\geq\dfrac{6}{5x+1}+2$
  4. $\sum\dfrac{1}{5x+1}\geq \dfrac{1}{2}$

[RIGHT][I][B]Nguồn: MathScope.ORG[/B][/I][/RIGHT]
 
tikita is offline   Trả Lời Với Trích Dẫn
The Following User Says Thank You to tikita For This Useful Post:
00000 (28-10-2012)
Old 15-10-2012, 10:20 PM   #3
silver soul
+Thành Viên+
 
Tham gia ngày: Aug 2011
Đến từ: KHTN
Bài gởi: 42
Thanks: 2
Thanked 38 Times in 20 Posts
Hớ hớ.... đề sư phạm năm nay.... khoai...
Mở hàng bài có vẻ dễ nhất.
Câu 3:a/ Ta chứng minh ${\rm{or}}{d_p}6 = {2^{n + 1}} $
Thật vậy ${6^{{2^{n + 1}}}} - 1 \vdots p;{6^{{2^n}}} - 1 \ne 0\left( {\bmod p} \right) $
Mặt khác ${6^{{2^{n + 1}}}} - 1 \ne 0\left( {\bmod {p^2}} \right) $
Theo bổ đề xuống thang ${v_p}\left( {{6^{p - 1}} - 1} \right) = {v_p}\left( {{6^{{\rm{or}}d}} - 1} \right) + {v_p}\left( {\frac{{p - 1}}{{{\rm{or}}d}}} \right) = 1 $
b/y' là nghịch đảo của y, khi đó ${\left( {xy'} \right)^2} \equiv 36\left( {\bmod p} \right) $
Vậy ta chỉ ra được một giá trị của k, đó là k=n-1
Ta chứng minh giá trị k này là duy nhất. Nếu có k>n-1 thỏa mãn thì
$0 \equiv {x^{{2^k}}} + {y^{{2^k}}} \equiv {y^{{2^k}}}\left( {{{\left( {xy'} \right)}^{{2^k}}} + 1} \right) \equiv {y^{{2^k}}}\left( {{6^{{2^{k + 1}}}} + 1} \right) \equiv 2{y^{{2^k}}} $ vô lí do tập xác định của y.
Nếu có k<n-1 thỏa mãn thì
$0 \equiv {x^{{2^{n - 1}}}} + {y^{{2^{n - 1}}}} \equiv {y^{{2^{n - 1}}}}\left( {{{\left( {xy'} \right)}^{{2^{n - 1}}}} + 1} \right) \equiv 2{y^{{2^{n - 1}}}} $
cũng vô lí nốt
vậy ta có dpcm
[RIGHT][I][B]Nguồn: MathScope.ORG[/B][/I][/RIGHT]
 
__________________
Chán đời cực độ..... Bờ sông nào cho ta.....
silver soul is offline   Trả Lời Với Trích Dẫn
Old 15-10-2012, 10:30 PM   #4
quykhtn
+Thành Viên+
 
Tham gia ngày: Mar 2012
Đến từ: Cái nôi của phở
Bài gởi: 259
Thanks: 78
Thanked 697 Times in 193 Posts
Trích:
Nguyên văn bởi Highschoolmath View Post
Câu 1: Cho các số thực dương $ x,y,z $ thỏa mãn $ x+y+z=3 $. Chứng minh rằng:
$$\frac{4x+5}{x^3+xy^2+3xyz}+\frac{4y+5}{y^3+yz^2+ 3xyz}+\frac{4z+5}{z^3+zx^2+3xyz} \geq \frac{162}{x^2+y^2+z^2+27} $$
Dù hình thức hơi cồng kềnh nhưng bài toán có thể chứng minh đơn giản như sau
Sử dụng bất đẳng thức Cauchy-Schwarz và giả thiết $ x+y+z=3 $ ta có
$$ \dfrac{4x+5}{x^3+xy^2+3xyz}+\dfrac{4y+5}{y^3+yz^2+ 3xyz}+\dfrac{4z+5}{z^3+zx^2+3xyz} $$
$$ =\dfrac{4+\dfrac{5}{x}}{x^2+y^2+3yz} + \dfrac{4+\dfrac{5}{y}}{y^2+z^2+3zx} + \dfrac{ 4 + \dfrac{5}{z}}{z^2+x^2+3xy} $$
$$ \geq \dfrac{4.9}{2(x^2+y^2+z^2)+3(xy+yz+zx)} + \dfrac{ 5 \left(\dfrac{1}{\sqrt{x}} + \dfrac{1}{\sqrt{y}} +\dfrac{1}{\sqrt{z}} \right)^2}{2(x^2+y^2+z^2)+3(xy+yz+zx)} $$
$$ \geq \dfrac{36+45}{2(x^2+y^2+z^2)+3(xy+yz+zx)}=\dfrac{1 62}{x^2+y^2+z^2+27} $$
[RIGHT][I][B]Nguồn: MathScope.ORG[/B][/I][/RIGHT]
 
__________________
The love make us weaker

Autumn
quykhtn is offline   Trả Lời Với Trích Dẫn
The Following 5 Users Say Thank You to quykhtn For This Useful Post:
boykhtna1 (16-10-2012), hephuongtrinh (29-10-2012), hieu1411997 (16-10-2012), K56khtn (16-10-2012), mathmath123 (29-10-2012)
Old 15-10-2012, 11:18 PM   #5
ratuno
+Thành Viên+
 
Tham gia ngày: Jun 2012
Bài gởi: 17
Thanks: 3
Thanked 8 Times in 6 Posts
Bài 2 $R_1, R_2$ là bán kính $(O_1),(O_2)$. $PA.PB = PD.PC \Leftrightarrow PA.PB = PE.PC \Leftrightarrow R_2^2-PO_2^2 = PO_1^2-R_1^2 \Leftrightarrow PO_1^2 +PO_2^2 = R_1^2 + R_2^2\Leftrightarrow PO = XO$
[RIGHT][I][B]Nguồn: MathScope.ORG[/B][/I][/RIGHT]
 
ratuno is offline   Trả Lời Với Trích Dẫn
The Following 3 Users Say Thank You to ratuno For This Useful Post:
dep_kom_n (16-10-2012), K56khtn (16-10-2012), tungk45csp (15-10-2012)
Old 15-10-2012, 11:20 PM   #6
quykhtn
+Thành Viên+
 
Tham gia ngày: Mar 2012
Đến từ: Cái nôi của phở
Bài gởi: 259
Thanks: 78
Thanked 697 Times in 193 Posts
Trích:
Nguyên văn bởi Highschoolmath View Post
Gõ lại cái đề cho tử tế:
Câu 1: Cho các số thực dương $ x,y,z $ thỏa mãn $ x+y+z=3 $. Chứng minh rằng:
$$ \frac{4x+5}{x^3+xy^2+3xyz}+\frac{4y+5}{y^3+yz^2+3x yz}+\frac{4z+5}{z^3+zx^2+3xyz} \geq \frac{162}{x^2+y^2+z^2+27} $$
Nhận xét
Bất đẳng thức chặt hơn sau vẫn đúng
Với $ x,y,z>0 ; x+y+z=3 $ ta có
$$ \dfrac{4x+5}{x^3+xy^2+3xyz}+\dfrac{4y+5}{y^3+yz^2+ 3xyz}+\dfrac{4z+5}{z^3+zx^2+3xyz} \geq \dfrac{27}{5} $$
[RIGHT][I][B]Nguồn: MathScope.ORG[/B][/I][/RIGHT]
 
__________________
The love make us weaker

Autumn
quykhtn is offline   Trả Lời Với Trích Dẫn
The Following 2 Users Say Thank You to quykhtn For This Useful Post:
hieu1411997 (16-10-2012), K56khtn (16-10-2012)
Old 15-10-2012, 11:24 PM   #7
tikita
Administrator

 
Tham gia ngày: Jun 2012
Bài gởi: 157
Thanks: 2
Thanked 84 Times in 53 Posts
Trích:
Nguyên văn bởi Highschoolmath View Post

Câu 4: Cho 2012 số thực $x_1,x_2,\ldots,x_{2012} $ đôi một khác nhau thõa mãn điều kiện $\sum_{i=1}^{2012}x_i=0 $. Chứng minh rằng tồn tại $C_{2012}^{502} $ bộ chỉ số $(i_1,i_2,\ldots,i_{503}) $ thõa mãn:
i/ $1 \leq i_1< i_2<\cdots<i_{503} \leq 2012 $
ii/ $x_{i_1}+x_{i_2} + \cdots + x_{i_{503}} \geq 0 $.
-------------------------------------Hết--------------------------------
Câu này hình như đề bị sai ấy!!
Ví dụ trong 2012 số này ta lấy đúng một số dương và 2011 số còn lại âm ( tổng vẫn bằng 0) thì số cách chọn bộ thỏa yêu cầu bài toán là $C_{2011}^{502}<C_{2012}^{502}$
[RIGHT][I][B]Nguồn: MathScope.ORG[/B][/I][/RIGHT]
 
tikita is offline   Trả Lời Với Trích Dẫn
Old 15-10-2012, 11:55 PM   #8
Mr Stoke
+Thành Viên Danh Dự+
 
Mr Stoke's Avatar
 
Tham gia ngày: Dec 2007
Bài gởi: 252
Thanks: 40
Thanked 455 Times in 95 Posts
Trích:
Nguyên văn bởi tikita View Post
Câu này hình như đề bị sai ấy!!
Ví dụ trong 2012 số này ta lấy đúng một số dương và 2011 số còn lại âm ( tổng vẫn bằng 0) thì số cách chọn bộ thỏa yêu cầu bài toán là $C_{2011}^{502}<C_{2012}^{502}$
Đề bài toán này ở trên bị đánh sai so với đề gốc, thay $502$ bởi $503$ thôi. Nói chung bài này khá dễ, có thể vì để là bài cuối cùng nên nhiều bạn bỏ lỡ cơ hội làm được.

Bài số học bạn trên chọn nhầm giá trị k, và quên chưa chứng minh nó là hợp số (sẽ mất 1.5 điểm nếu không chứng minh điều này).

Bài Bất đẳng thức có cách: sử dụng AM-GM trực tiếp kết hợp bất đẳng thức kiểu
$(a+x)(a+y)(a+z)\geq(a+\sqrt[3]{xyz})^3$ là xong.

Bài hình học khá tầm thường, chủ yếu cái hình vẽ khiến nhiều bạn hs không nhìn ra.
[RIGHT][I][B]Nguồn: MathScope.ORG[/B][/I][/RIGHT]
 
Mr Stoke is offline   Trả Lời Với Trích Dẫn
The Following 3 Users Say Thank You to Mr Stoke For This Useful Post:
MathForLife (16-10-2012), mathmath123 (16-10-2012), tungk45csp (16-10-2012)
Old 16-10-2012, 12:04 AM   #9
Highschoolmath
Moderator
 
Highschoolmath's Avatar
 
Tham gia ngày: Apr 2008
Đến từ: Hàm Dương-Đại Tần
Bài gởi: 698
Thanks: 247
Thanked 350 Times in 224 Posts
Trích:
Nguyên văn bởi Mr Stoke View Post
Đề bài toán này ở trên bị đánh sai so với đề gốc, thay $502$ bởi $503$ thôi. Nói chung bài này khá dễ, có thể vì để là bài cuối cùng nên nhiều bạn bỏ lỡ cơ hội làm được.

Bài số học bạn trên chọn nhầm giá trị k, và quên chưa chứng minh nó là hợp số (sẽ mất 1.5 điểm nếu không chứng minh điều này).

Bài Bất đẳng thức có cách: sử dụng AM-GM trực tiếp kết hợp bất đẳng thức kiểu
$(a+x)(a+y)(a+z)\geq(a+\sqrt[3]{xyz})^3$ là xong.

Bài hình học khá tầm thường, chủ yếu cái hình vẽ khiến nhiều bạn hs không nhìn ra.
Cái đề mấy đứa chụp ảnh rồi gửi cho em nên mờ lắm, em không dịch được, toàn phải đoán, thầy có thể cho em xin cái đề chuẩn của bài 4 được không ạ?
PS: Dù thay 502 bởi 503 thì theo lý luận của bạn titika ở trên, $C_{2011}^{502}<C_{2012}^{502}<C_{2012}^{503}$, đề vẫn chưa chính xác mà thầy.
[RIGHT][I][B]Nguồn: MathScope.ORG[/B][/I][/RIGHT]
 
__________________
As long as I live, I shall think only of the Victory......................

thay đổi nội dung bởi: Highschoolmath, 16-10-2012 lúc 12:08 AM
Highschoolmath is offline   Trả Lời Với Trích Dẫn
Old 16-10-2012, 12:34 AM   #10
Mr Stoke
+Thành Viên Danh Dự+
 
Mr Stoke's Avatar
 
Tham gia ngày: Dec 2007
Bài gởi: 252
Thanks: 40
Thanked 455 Times in 95 Posts
MS xin lỗi, lúc này MS không nhớ rõ đề gốc chính xác là số $\binom{2011}{502}$ hay $\binom{2012}{503}$. Nhưng theo MS nghĩ, bài toán muốn đúng thì số đó chính xác phải là $\binom{2011}{502}$. Còn số $\binom{2012}{503}$ là sai. Để mai MS sẽ xem lại đề gốc viết số nào vì MS cũng chỉ nhớ mang máng đề nó thế.

Bài toán tổng quát như sau: Cho $m\geq2,n\geq1$ là các số nguyên thỏa mãn $n\mid m$ và $m$ số thực $x_1,\ldots,x_m$ có tổng bằng $0$. Chứng minh rằng có thể chỉ ra ít nhất $\binom{m-1}{n-1}$ bộ chỉ số $1\leq i_1<\cdots<i_n\leq m$ mà tổng $x_{i_1}+\cdots+x_{i_n}\geq0$.

Gửi tạm file này, MS tạm bỏ phần lời giải đề ngày 1, để các bạn thảo luận.
[RIGHT][I][B]Nguồn: MathScope.ORG[/B][/I][/RIGHT]
 
File Kèm Theo
Kiểu File : pdf desupham2012.pdf (163.7 KB, 341 lần tải)
Mr Stoke is offline   Trả Lời Với Trích Dẫn
The Following 2 Users Say Thank You to Mr Stoke For This Useful Post:
MathForLife (16-10-2012), pexea12 (16-10-2012)
Old 28-10-2012, 10:07 AM   #11
Highschoolmath
Moderator
 
Highschoolmath's Avatar
 
Tham gia ngày: Apr 2008
Đến từ: Hàm Dương-Đại Tần
Bài gởi: 698
Thanks: 247
Thanked 350 Times in 224 Posts
Trích:
Nguyên văn bởi Mr Stoke View Post
MS xin lỗi, lúc này MS không nhớ rõ đề gốc chính xác là số $\binom{2011}{502}$ hay $\binom{2012}{503}$. Nhưng theo MS nghĩ, bài toán muốn đúng thì số đó chính xác phải là $\binom{2011}{502}$. Còn số $\binom{2012}{503}$ là sai. Để mai MS sẽ xem lại đề gốc viết số nào vì MS cũng chỉ nhớ mang máng đề nó thế.

Bài toán tổng quát như sau: Cho $m\geq2,n\geq1$ là các số nguyên thỏa mãn $n\mid m$ và $m$ số thực $x_1,\ldots,x_m$ có tổng bằng $0$. Chứng minh rằng có thể chỉ ra ít nhất $\binom{m-1}{n-1}$ bộ chỉ số $1\leq i_1<\cdots<i_n\leq m$ mà tổng $x_{i_1}+\cdots+x_{i_n}\geq0$.

Gửi tạm file này, MS tạm bỏ phần lời giải đề ngày 1, để các bạn thảo luận.
Lữ với cả Kien10a1 vào tập trung giải nốt bài tổ hợp này đi các bạn. Bài này mình chỉ chứng minh được ít nhất có $\frac{(n-k)!}{[(m-1)!]^k} $ bộ như vậy thôi ($k=\frac{n}{m} $), so với chặn dưới $C^{m-1}_{n-1} $ của bài toán thì kết quả của mình yếu hơn rất nhiều.
[RIGHT][I][B]Nguồn: MathScope.ORG[/B][/I][/RIGHT]
 
__________________
As long as I live, I shall think only of the Victory......................

thay đổi nội dung bởi: Highschoolmath, 28-10-2012 lúc 10:12 AM
Highschoolmath is offline   Trả Lời Với Trích Dẫn
Old 28-10-2012, 10:40 AM   #12
giga19111996
+Thành Viên+
 
Tham gia ngày: Oct 2012
Bài gởi: 12
Thanks: 5
Thanked 2 Times in 2 Posts
Bài 2 thì dùng trục đẳng phương với công thức tính độ dài trung tuyến thì có thể thu ngay kết quả
[RIGHT][I][B]Nguồn: MathScope.ORG[/B][/I][/RIGHT]
 
giga19111996 is offline   Trả Lời Với Trích Dẫn
Old 29-10-2012, 06:50 AM   #13
Traum
Moderator
 
Traum's Avatar
 
Tham gia ngày: Nov 2007
Đến từ: cyber world
Bài gởi: 413
Thanks: 14
Thanked 466 Times in 171 Posts
Trích:
Nguyên văn bởi Highschoolmath View Post
Lữ với cả Kien10a1 vào tập trung giải nốt bài tổ hợp này đi các bạn. Bài này mình chỉ chứng minh được ít nhất có $\frac{(n-k)!}{[(m-1)!]^k} $ bộ như vậy thôi ($k=\frac{n}{m} $), so với chặn dưới $C^{m-1}_{n-1} $ của bài toán thì kết quả của mình yếu hơn rất nhiều.
Bài toán tổng quát là một hệ quả của định lý Baranyai [Only registered and activated users can see links. ]

Do $n|m $ nên ta có thể chia $m $ số vào $m/n $ bộ gồm $n $ số. Và hiển nhiên tổng của tổng các bộ số = 0, nên tồn tại ít nhất một bộ $n $ số có tổng không âm.

Chú ý 1: $\binom{m-1}{n-1} = \frac{n}{m}\binom{m}{n} $.

Chú ý 2: có tất cả $\binom{m}{n} $ bộ $n $ số.

Chú ý 3: bài toán được giải quyết nếu ta chia được $\binom{m}{n} $ bộ $n $ số thành $\frac{n}{m}\binom{m}{n} $ nhóm sao cho mỗi bộ thuộc vào đúng một nhóm, và hai bộ thuộc cùng một nhóm thì không có phần tử chung. Đây chính là nội dung của định lý Baranyai.

Trở lại bài toán của đề SP. Đó là một trường hợp riêng với $m=4n $. Đối với $m = 4n $ thì có một cách đơn giản hơn như sau:

Bài giải:

Nhận xét 1: Với $2k $ số thực có tổng không âm thì có thể chọn ra $\frac{1}{2}\binom{2k}{k} $ bộ $k $ số có tổng không âm.

Dễ thấy nhận xét 1 đúng vì một trong hai bộ $k $ và bộ bù của nó sẽ có ít nhất một bộ có tổng không âm.

Áp dụng nhận xét vào bài toán ta có:

Có ít nhất $\frac{1}{2}\binom{4n}{2n} $ bộ $2n $ số có tổng không âm. Với mỗi bộ $2n $ số có tổng không âm, cho ta $\frac{1}{2}\binom{2n}{n} $ bộ $n $ số có tổng không âm. Vậy tổng cộng ta có $\frac{1}{4}\binom{4n}{2n}\binom{2n}{n} $ bộ $n $ số có tổng không âm (có thể trùng nhau). Tiếp theo ta sẽ tính xem mỗi bộ $n $ số có tổng không âm được tính lặp bao nhiêu lần. Dễ thấy là mỗi bộ $n $ số có tổng không âm sẽ thuộc về nhiều nhất là $\binom{3n}{n} $ bộ $2n $ số có tổng không âm. Do đó mỗi bộ $n $ số bị đêm lặp không quá $\binom{3n}{n} $ lân. Số bộ có tổng không âm khác nhau ít nhất là $\frac{1}{4}\binom{4n}{2n}\binom{2n}{n}/\binom{3n}{n} = \frac{1}{4}\binom{4n}{n} $. ĐPCM.
[RIGHT][I][B]Nguồn: MathScope.ORG[/B][/I][/RIGHT]
 
__________________
Traum is giấc mơ.

thay đổi nội dung bởi: Traum, 29-10-2012 lúc 06:52 AM
Traum is offline   Trả Lời Với Trích Dẫn
The Following 2 Users Say Thank You to Traum For This Useful Post:
kien10a1 (29-10-2012), nghiepdu-socap (29-10-2012)
Old 29-10-2012, 12:05 PM   #14
Highschoolmath
Moderator
 
Highschoolmath's Avatar
 
Tham gia ngày: Apr 2008
Đến từ: Hàm Dương-Đại Tần
Bài gởi: 698
Thanks: 247
Thanked 350 Times in 224 Posts
Trích:
Nguyên văn bởi Traum View Post
Bài toán tổng quát là một hệ quả của định lý Baranyai [Only registered and activated users can see links. ]

Do $n|m $ nên ta có thể chia $m $ số vào $m/n $ bộ gồm $n $ số. Và hiển nhiên tổng của tổng các bộ số = 0, nên tồn tại ít nhất một bộ $n $ số có tổng không âm.

Chú ý 1: $\binom{m-1}{n-1} = \frac{n}{m}\binom{m}{n} $.

Chú ý 2: có tất cả $\binom{m}{n} $ bộ $n $ số.

Chú ý 3: bài toán được giải quyết nếu ta chia được $\binom{m}{n} $ bộ $n $ số thành $\frac{n}{m}\binom{m}{n} $ nhóm sao cho mỗi bộ thuộc vào đúng một nhóm, và hai bộ thuộc cùng một nhóm thì không có phần tử chung. Đây chính là nội dung của định lý Baranyai.

Trở lại bài toán của đề SP. Đó là một trường hợp riêng với $m=4n $. Đối với $m = 4n $ thì có một cách đơn giản hơn như sau:

Bài giải:

Nhận xét 1: Với $2k $ số thực có tổng không âm thì có thể chọn ra $\frac{1}{2}\binom{2k}{k} $ bộ $k $ số có tổng không âm.

Dễ thấy nhận xét 1 đúng vì một trong hai bộ $k $ và bộ bù của nó sẽ có ít nhất một bộ có tổng không âm.

Áp dụng nhận xét vào bài toán ta có:

Có ít nhất $\frac{1}{2}\binom{4n}{2n} $ bộ $2n $ số có tổng không âm. Với mỗi bộ $2n $ số có tổng không âm, cho ta $\frac{1}{2}\binom{2n}{n} $ bộ $n $ số có tổng không âm. Vậy tổng cộng ta có $\frac{1}{4}\binom{4n}{2n}\binom{2n}{n} $ bộ $n $ số có tổng không âm (có thể trùng nhau). Tiếp theo ta sẽ tính xem mỗi bộ $n $ số có tổng không âm được tính lặp bao nhiêu lần. Dễ thấy là mỗi bộ $n $ số có tổng không âm sẽ thuộc về nhiều nhất là $\binom{3n}{n} $ bộ $2n $ số có tổng không âm. Do đó mỗi bộ $n $ số bị đêm lặp không quá $\binom{3n}{n} $ lân. Số bộ có tổng không âm khác nhau ít nhất là $\frac{1}{4}\binom{4n}{2n}\binom{2n}{n}/\binom{3n}{n} = \frac{1}{4}\binom{4n}{n} $. ĐPCM.
Links die rồi anh ạ.
Về bài toán tổng quát, em chuyển nó sang bài toán đếm này mà không sao giải nổi:
Có $k.m $ ($k,m \in N^* $) cái kẹo đôi một phân biệt được chia cho $k $ đứa trẻ sao cho mỗi đứa trẻ có đúng $m $ chiếc kẹo. Giả sử ta có $T $ cách chia kẹo như trên sao cho nếu gọi $a(i,j) $ là tập các chiếc kẹo được chia cho em thứ $i $ tại lần chia thứ $j $, thì tất cả các $a(i,j) $ đều đôi một phân biệt ($\forall 1 \leq i \leq k; 1 \leq j \leq T $). Tìm giá trị lớn nhất của $T $.
[RIGHT][I][B]Nguồn: MathScope.ORG[/B][/I][/RIGHT]
 
__________________
As long as I live, I shall think only of the Victory......................
Highschoolmath is offline   Trả Lời Với Trích Dẫn
Old 29-10-2012, 02:33 PM   #15
kaka_ak9
+Thành Viên+
 
kaka_ak9's Avatar
 
Tham gia ngày: Apr 2012
Bài gởi: 34
Thanks: 16
Thanked 2 Times in 2 Posts
Trích:
Nguyên văn bởi silver soul View Post
Hớ hớ.... đề sư phạm năm nay.... khoai...
Mở hàng bài có vẻ dễ nhất.
Câu 3:a/ Ta chứng minh ${\rm{or}}{d_p}6 = {2^{n + 1}} $
Thật vậy ${6^{{2^{n + 1}}}} - 1 \vdots p;{6^{{2^n}}} - 1 \ne 0\left( {\bmod p} \right) $
Mặt khác ${6^{{2^{n + 1}}}} - 1 \ne 0\left( {\bmod {p^2}} \right) $
Theo bổ đề xuống thang ${v_p}\left( {{6^{p - 1}} - 1} \right) = {v_p}\left( {{6^{{\rm{or}}d}} - 1} \right) + {v_p}\left( {\frac{{p - 1}}{{{\rm{or}}d}}} \right) = 1 $
b/y' là nghịch đảo của y, khi đó ${\left( {xy'} \right)^2} \equiv 36\left( {\bmod p} \right) $
Vậy ta chỉ ra được một giá trị của k, đó là k=n-1
Ta chứng minh giá trị k này là duy nhất. Nếu có k>n-1 thỏa mãn thì
$0 \equiv {x^{{2^k}}} + {y^{{2^k}}} \equiv {y^{{2^k}}}\left( {{{\left( {xy'} \right)}^{{2^k}}} + 1} \right) \equiv {y^{{2^k}}}\left( {{6^{{2^{k + 1}}}} + 1} \right) \equiv 2{y^{{2^k}}} $ vô lí do tập xác định của y.
Nếu có k<n-1 thỏa mãn thì
$0 \equiv {x^{{2^{n - 1}}}} + {y^{{2^{n - 1}}}} \equiv {y^{{2^{n - 1}}}}\left( {{{\left( {xy'} \right)}^{{2^{n - 1}}}} + 1} \right) \equiv 2{y^{{2^{n - 1}}}} $
cũng vô lí nốt
vậy ta có dpcm
Anh có thề nói kĩ hơn về bổ đề xuống thang anh dùng ở câu a được ko ạ
[RIGHT][I][B]Nguồn: MathScope.ORG[/B][/I][/RIGHT]
 
kaka_ak9 is offline   Trả Lời Với Trích Dẫn
The Following User Says Thank You to kaka_ak9 For This Useful Post:
ohmymath (10-06-2013)
Trả lời Gởi Ðề Tài Mới

Bookmarks


Quuyền Hạn Của Bạn
You may not post new threads
You may not post replies
You may not post attachments
You may not edit your posts

BB code is Mở
Smilies đang Mở
[IMG] đang Mở
HTML đang Tắt

Chuyển đến


Múi giờ GMT. Hiện tại là 12:30 PM.


Powered by: vBulletin Copyright ©2000-2024, Jelsoft Enterprises Ltd.
Inactive Reminders By mathscope.org
[page compression: 119.04 k/135.87 k (12.38%)]